Matematică, întrebare adresată de lilicole, 8 ani în urmă

Ce formula are?????????????????????

Anexe:

GreenEyes71: Ești sigură că sub suma este n³ ? Nu cumva este k³ ?
lilicole: Greseala mea
lilicole: Te rog ma ajuti la intrebarea pe care am pus o acum

Răspunsuri la întrebare

Răspuns de GreenEyes71
2

Salut,

\sum\limits_{k=1}^nk^3=\dfrac{n^2(n+1)^2}4.

Green eyes.


lilicole: Multuuumesc
Alte întrebări interesante